Sie sind auf Seite 1von 16

SOLUTIONS FOR QUANT QUESTIONS THAT HAVE

APPEARED IN CAT IN THE LAST 3 YEARS


Solutions for question 1: N2 − 5N + 6
Now all these will be non-mates of the
1. Least element of I = 1000. 2
Greatest element of I = 5000. member (a, b) Choice (4)
Let the required number be N. 6. Let (a, b) and (b, c) be two mates in X
N = Number of integers from 1000 to 5000 which can They have element b in common.
be formed with 0 or 1 or 2 or 3 or 5. Now all the members which have b as one of the
If the integers are less than 5000, the thousands digit has elements will be a common mate to both (a, b) and
three possibilities (1 or 2 or 3). Each of the remaining (b, c). Other than a, b and c, there are. N – 3 elements.
digits has five possibilities 0, 1, 2, 3 or 5 Now with each of these n – 3 elements with element b
∴ A total of (3) (5) (5) (5) or 375 integers can be formed can form a common mate.
less than 5000. In addition the member (a, c) will also be a common mate
The only integer not less than 5000 is 5000 itself. to both the members (a, b) and (b, c).
∴ N = 375 + 1 = 376 Choice (1) So, total number of common mates = N – 3 + 1
= N – 2. Choice (1)
Solutions for questions 2 and 3:
Solutions for questions 7 to 42:
Any shortest path from H to O must include AB.
∴ The number of shortest paths from H to O is mn, where m 7. If the sum of the digits of a number is divisible by 9,
is the number of shortest paths from H to A and n is the then the number is divisible by 9.
number of shortest paths from B to O Sum of the given digits is 0 + 1 + 2 + 4 + 6 + 8 + 9 = 30.
m = 4C2 = 6 and n = 7C2 = 21. We leave two digits such that the sum of the other five
∴ mn = 126. digits is multiple of 9.
0, 4, 6, 8, 9 or 0, 1, 2, 6, 9 the number of arrangements
H C in each case 4! 41 hence 2.4.4! = 192. Choice (B)

8. If the last two digits are divisible by 4, then the number


A
is divisible by 4.
R P
Given digits is 0, 2, 4, 5, 6, 7.
The possible two digits are 04, 20, 40, 60, 24, 52, 56,
B
64, 72, 76
O ∴ The number of five digit number formed which are
end with 04 is _ _ _ 04 = 4P3.
c 13 7 1
∴ The number of five digit numbers formed which are
6 6 1 end with 04 or 20, 40 or 60 = 4 × P3 = 96.
4
5 1 The number of five digit number formed which are end
P 4
1 with 24 is _ _ _ 24 = 3 × 3 × 2 = 18
3 ∴The number of five digit number formed which are
1
2 end with 24, 52, 56, 64, 72 or 76 = 6 × 18 = 108.
1 1
∴ Hence the total number of five digit number which
O
are divisible by 4 can be formed using the given digits
In the figure above, the number at a node indicates the = 96 + 108 = 204 Choice (C)
number of shortest paths to reach that node from O.
9. All players who play the same game are treated as one
2. mn = 126 Choice (4) unit. Then four units can be arranged in a row in 4!
ways.
3. There are 126 shortest paths from H to O and Now the players in each game can be arranged among
13 shortest paths from O to C. themselves in 6! 7! 8!and 9! ways respectively.
∴There are a total of (126) (13) or 1638 shortest paths The required number of arrangements = 4!6!7!8!9!
from H to C via O. Choice (1) Choice (D)

Solutions for question 4: 10. Each student can be sent into any of the five sections.
15
∴ Seven students can be sent 5 × 5 × 5 × 5 × 5 × 5 × 5
7
4. The terms in the expansion of (p + q + r) have the i.e 5 ways. Choice (B)
x y z
form Kp q r where x + y + z = 15. The number of terms
is the number of non-negative integral solutions of 11. The sum of the all the five digit numbers formed by the
x + y + z = 15, which is equal to the number of positive digits a, b, c, d, e without repetition in 4! (a + b + c + d
integral solutions of x + y + z = 18, which is 17C2 or 136. + e) (11111).
Choice (1) The sum of the all the five digit number formed by the
digits 2, 4, 6, 7, 8 is 4! (2 + 4 + 6 + 7 + 8) (11111)
Solutions for questions 5 and 6: = 7199928. Choice (B)

5. Let (a, b) be a member of X. 12. Since 40 lines are given, The number of points of
Now non mates of this member will have any of the intersection of these 40 lines is 40C2. But 2 lines are
remaining elements other than a and b. concurrent they are intersected at only one point.
Remaining elements in N other than a and be is N – 2 Similarly, 15 lines are concurrent they intersect at only
one point. 13 lines are parallel.
(N − 2) (N − 3) N2 = 5N + 6
elements = N – 2 C2 = = They do not intersect.
2 .1 2 Hence required number of points of intersections
= 40C2 – 66 – 105 – 78 + 2 = 533. Choice (C)

1
13. Let E = (x + y – z)100 Alternative Method:
n a b c
The general term in the expansion is C1 x y z , where
a + b + c = 100. Ar of the double trapezium
For the terms which are negative c is odd, i.e., c = 1, 3, Ar of the two isosceles ∆
5, … 99.
∴ a + b = 99, 97, --- 1 -------(A) =
Ar XYPQ
The number of negative terms is equal to the sum of the Ar PMY + Ar QNX
number of non-negative integral solutions of the fifty
equations denoted by (A) which is 100C1 + 98C1 + …. 2C1 Ar QNYT NY 3
= = = =2 3 +3
= 100 + 98 + ….. + 2 Ar TYMP YM 2− 3
(50) (51) Choice (2)
= 2 = 2550. Choice (C)
2
17.
1 A
14. Area of triangle PQR = (PS) (QR)
2
Let the circumradius of ∆PQR be R1 for a triangle with
abc C1 • 120° • C2
sides a, b, c. Then area of the triangle =
4R1
1
(PS (QR)
2 B
=
(PQ) (PR ) (QR ) P
Let C1 and C2 be the centers of the circles and A and B
4R1 19.5 be the intersection points common radius for both
11
1 5 circles
⇒ (5) (QR) ∆ C1 C2 A is equilateral.
2 Q R
S Similarly ∆ C1 C2 B is equilateral.
=
(19.5) (11) (QR ) ∴∠AC1C2 = ∠BC1C2 = 60°
4R1 ∴∠AC1B = 120°
⇒ R = 21.45 Choice (4) Required area = 2(area of the segment AC2B)
Area of segment AC2B = Area of the sector C1AC2
15. Let AB = 7, BC = 16 and CA = y (where y is an integer) B – Area of triangle C1C2A
As ABC is an obtuse triangle, the square of its longest 120 1
side must be greater than the sum of the squares on = π(2)2 – (C1A) (CB) Sin ∠ACB
360 2
the other two sides.
4π 1  4π 
Max(7, 16, y) = 16 or y. = – (2)(2) sin120° =  − 3  sq.cm
If Max = 16, 162 > 72 + y2. 3 2  3 
∴ y2 < 207  8π 
i.e., y ≤ 14. ∴ Required area =  − 2 3  sq.cm.
By triangle inequality y + 7 > 16 i.e., y > 9.  3 
∴y = 10, 11, 12, 13 or 14. Choice (3)
If Max = y, y2 > 162 + 72.
∴ y > 305 ie., y ≥ 18.
2 18. Triangles PUT and P
By triangle inequality, 16 + 7 > y i.e., y < 23. PVR are similar.
⇒ y = 18, 19, 20, 21 or 22. PU PV
∴ = S
∴ Total number of possibilities of y is 10. Choice (2) UT VR U T
21 7
16. T Q = =
P 6 2
Let UT = x cm v
Q R
7
PU = x cm
30° 2
M Y N
30° X 7x
UV = (21 – ) cm
2
Total surface area of the cylinder
S 7x
R = (2πx) (x + 21 – ) cm2= πx(42 – 5x) cm2
2
Let QN = 1.
π
∴ NY = 3 (Similarly MX = 3 ) and PQ = 2 = (5x)(42 – 5x) cm2
5
∴ MY = PQ – NY = 2 – 3 (Similarly NX = 2 – 3 ) As the sum of the two factors 5x and 42 – 5x is
constant, the product is maximum when 5x = 42 – 5x or
Ar (∆XQR) + Ar (∆YPS) = 2(2 – 3 ) 21
x=
= 4 – 2 3 Ar(PQRS) = 4. 5.
The corresponding area is
Ar of PQ X RSY 4 − ( 4 − 2 3) 2 3
∴ = =  21   42 + 210 147 
Re maining area 2(2 − 3) 2(2 − 3) 2π     cm
2

 5   10 
3 441π
= = 3 (2 + 3 ) = 2 3 + 3 = cm2 Choice (5)
2− 3 5

2
19. The two extreme cases are shown below. 22. A

8 3
X 8 3
M• •N
Y 60°

B D C
∆ABC is equilateral such that radius = 8 3 cm
∠XMN = 0°
X 8 3
BD = =16.
sin 60°
∴ side of equilateral triangle = 32 cm. Choice (B)
M• •N
23. Let the radius of each A
circle be r.
AB = AG + GI + IB.
Y
∠A = 60°
∠XMN = 60° ∠A
G
D H
∴ 0° ≤ θ ≤ 60° Choice (3) ∴ ∠GAD = = 30°. •
2
20. GD I
AG = =r 3 . E
D tan ∠GAD • •F
Similarly IB = r 3 . B C
x y y x GI = DE = sum of the
A• F •B radii of the circles whose centres are D and E = 2r.
C G
∴ AB = r 3 + 2r + r 3 = 2(1 + 3 )r.
E

AB
r
(
= 2 1+ 3 ) Choice (C)
Let A and B be the centres of the circles. Let D and E
be the points of intersection of the two circles. Let C be 24. Number of triangles which can be drawn by using the
the intersection of AB and CD. N points as vertices = NC3.
Let AF = x, FC = CG = y and GB = x. Number of triangles which can be drawn by using all but
Radius of each circle = x + 2y. one of the N points as vertices = N–1C3.
x + 2y 1 x + 2y 2 N
C3 – N–1C3 = 210.
= i.e. = .
x + 2y + x 3 x+y 3 N(N − 1)(N − 2) (N − 1)(N − 2)(N − 3 )
− = 210
CB x+y 3 6 6
cos∠CBD = = = . (N – 1) (N – 2) = 420
BD x + 2y 2
N2 – 3N – 418 = 0
π (N – 22) (N + 19) = 0
∴ ∠CBD = .
6 As N > 0, so N ≠ –19.
Common region area = 2 (Area of sector BED – Area of ∴ N = 22. Choice (B)
 2π  25. Let the sides of the triangle be a, b and c. Let its area
 
.π(BD) − (BD) sin ∠EBD 
 6 2 1 2 be A.
∆BED) = 2
 2π 2  X = abc
 
   a + b + c   abc  A abc
Y =   .   . =
π 3  π  2   4 A   a + b + c  4
= 2 BD 2  − . [∵ ∠EBD = ]  
6 4  3  2 
 
Y 1
  ∴ = Choice (A)
(2) BD2  π − 3 

X 4
6 4  2π − 3 3
∴ Required ratio = = . 26.
P T Q
π BD 2 6π U9 32
Choice (D) V
(PQ – 32)
21. Number of equilateral triangles which can be formed of
side a cm using equilateral triangles of side b cm (PQ – 9)
2
a
=  
b S W R
∴ Total number of triangles used PQRS is a square.
 16 
2
 18 
2
 30 
2 ∴ ∠S = 90° and PQ = QR
=   +   ...... 2 2
 = 8 + 9 + ……15
2
= RS = PS
 2   2   2  SU2 = UW2 + SW2
= 12 + 22 + …….. 152 – (12 + 22 + ….. 72) ⇒ PQ2 = (PQ – 9)2 + (PQ – 32)2 [∵ PQ = SU]
Considering PQ = x, we get
=
(15)(16)(31) − (7)(8)(15) = 1100. ⇒ x – 82x + 1105 = 0
2

6 6 ⇒ (x – 17) (x – 65) = 0
Choice (B) ∴ x = 65 or 17
As x > 32, ∴ x = 65. Choice (B)
3
27. 30. The sides given are 68 and 32.
68 = 4 × 17; 32 = 4 × 8
A
As 8, 15, 17 from an right angled triangle, the third side
8 8 = 4 × 15 = 60 Choice (D)

B• •C 31. Let F and G be the midpoints on the diameters of the


cylinder.
Required volume = Volume of the cone ABC
D 1 A
= π(FC)2(AF)
Let B and C be the centres of the circles. 3
Let A and D be the points of intersection. AF FC F
AB = AC = 8 cm. = .
AG GE B C
180° − ∠A
∴ ∠B = ∠C = = 45° AF 15
= .
2 54 18 G
∴ ∠ABD = 90°. AF = 45. D E
Area of the shaded region = 2(Area of the sector BAD –
1
 90°  ∴ Required volume = π
.π(BA )2 − .(BA )(BD )
1
Area of ∆BAD) = 2  3
 360° 2  2
(15) (45) = 3375π cubic cm. Choice (D)
π  π−2 2
=  − 1(BA )2 = (8) = 32(π – 2) sq cm. 32. Let the sides of the cubes be a cm and b cm respectively.
2  2 4a2 + 4b2 = 468 and a3 + b3 = 945
Choice (D) a2 + b2 = 117 and a3 + b3 = 945 → (1)
Let a be ≥ b.
28. Then a3 ≥ b3.
A E B (1) ⇒ 2a3 ≥ 945 ⇒ a3 ≥ 472.5. Also a3 < 945. → (1)
a is an integer. ∴ a3 is a perfect cube
(1) ⇒ a3 = 512 or 729.
P
a3 = 512 ⇒ b3 = 433. a3 = 729 ⇒ b3 = 216.
b is an integer. ∴ b3 is a perfect cube.
F ∴ b3 = 216 and a3 = 729 i.e., b = 6 and a = 9.
2 3375(2)
Volume of the hemisphere = π (9 + 6)3 = π
3 3
C D = 2250π cubic cm. Choice (B)
Area of region P = 2(Area of sector EFB – Area of A
∆EFB) 33.
 90° 
.π(EB )2 − .EF.EB 
1 12
=2 
 360° 2  40
32
π  π−2 2
=  − 1(EB )2 = (8) 20
 2  2
20
Therefore the area of field accessible to the cows E C
 π(8 )2  B 4
= 2 24
 – Area of region P
 2  322 = 1024, 242 = 576 and 402 = 1600.
  π(8 )2  π−2 2 322 + 242 = 402.
∴ Required area = AB2 –  2 − (8 )  ∴ The triangle is right angled at B.
  2   2  
    Circumradius of ∆ABC =
AC
= 20.
2 2
= 16 – (64π – (π – 2) 32)
= 256 – 64π + 32π – 64 AD : EC = 12 : 4 = 3 : 1. Choice (A)
= (192 – 32π) sq m. Choice (C)
34. Let the number of 1-femto, 2-femto and 20-femto notes
P with him be x, y and z respectively.
29. x + 2y + 20z = 47
θ (1) (3) (2)
(3) (2)
A
• (5) (1)
(7) (0)
30 -------------------------------------
(1) (13) (1)
α θ α (3) (12)
Q
R B ⋮ ⋮
The figure above is not to scale. (27) (0)
In ∆PQA and ∆QAR, PQ = QR. -------------------------------------
PA = AR (∵ QA is the median to PR) (1) (23) (0)
QA is a common side ⋮
∴ ∆PAQ ≅∆QAR. (47) (0)
--------------------------------------
∴ ∠PQA = ∠AQB i.e., ∠AQB = 30°. We can see that there are 4 + 14 + 24 or 42 ways to
∠ABQ = ∠AQB = 30° (∵ AQ = AB). Choice (B) settle the bill. Choice (5)
4
35. Let the amount on the cheque be Rs.x.y. 39. Let the bag contain x two rupee coins and y five rupee
Amount received by Mohan = Rs.y.x coins
Amount left with Mohan after he bought the biscuit Let 2x + 5y = k → (1)
= Rs.(y.x – 1.50) 75
4x.y = y.x – 1.50 → (1) Again given 2x + 5(3y) = k + k
100
In Rs.x.y, y represents the number of paise.
7k
∴ 0 < y ≤ 99 ⇒ 2x + 15y = → (2)
In Rs.y.x, x represents the number of paise 4
0 < x ≤ 99 (2) – (1) given
∴ 0 < y, x ≤ 99 → (2) 3k
10y =
(1) ⇒ 4(100x + y) = 100y + x – 150 4
⇒ 399x = 96y – 150 3k
⇒ 133x = 32y – 50 y=
40
⇒ 128x + 5(x + 10) = 32 → (3)
when k is multiple of 40 then y is multiple of 3
i.e., 5(x + 10) = 32(y – 4x) when k = 240
∴ x + 10 must be divisible by 32. y = 18
(2) ⇒ x + 10 = 32 or 64 or ∴ The number of five rupee coins may be 18.
96 i.e., x = 22 or 54 or 86. Choice (C)
If x = 22, (3) ⇒ y = 93
5(x + 10 ) 40. Say Abilash has answered x questions correctly and y
(3) ⇒ y = 4x +
32 questions wrongly.
∴ y > 4x. x + y > 85 → (1)
4x – 2y = 70
(2) ⇒ 99 > y > 4x i.e.,
2x – y = 35
x < 24.75.
2x = y + 35
∴ x = 22 is the only possibility.
x + y > 85 ⇒ 2x + 2y > 170
∴ y = 93
y + 35 + 2y > 170
∴ y.x = 93.22. The amount left with him after he bought 3y > 135; y > 45
the biscuit is 91.72. Choice (3) when y is an even number then the number of marks
obtained by Abhilash cannot be 70.
1  m ∴y is always odd which is greater then 45 If y = 47
36. After A gives to B, he has = m –  m − 1 = + 1
2  2 4x – 2y = 70
After giving to C, he has 4x = 70 + 94
1m 164
m   m 3 x= ; x = 41
+ 1 –   + 1 − 1 = + 4
2  2  2   4 2
∴ The minimum number of questions that Abhilash
After giving to D, he has answered correctly is 41. Choice (D)
m 3 1 m 3   m 7 m 7
+ –   +  − 1 = + ⇒ + =4 41. Let Arun, Varun, Kiran have initially Rs.a, Rs.b,
4 2  
2 4 2   8 4 8 4 Rs.c respectively
⇒ m = 18 Choice (5) follows. Choice (5) Given a + b + c = 8000
Let after spending their amount they have Rs.x, Rs.y,
1 Rs.z respectively
37. Given a ∝ b ∝c2
b Given x : y : z = 14 : 22 : 15
ab = k1 b = k2c2 x = 14 k; y = 22k; z = 15k
b = 3, a = 4, ⇒ k1 = 12 (x + 50) + y + 100z + 200 = 8000
Given b = 3 when c = 6 x + y + z = 7650
3 = k2 : 36 ⇒ k2 = 1/12 14 k + 22k + 15 k = 7650
51 k = 7650
k
ab = k1 ⇒ a = 1 7650
b k= = 150
51
12 4
a= = ∴y = 22k
27 9 = 22 × 150 = 3300
b = k2 c ⇒ c2 =
2 b
=
27 ∴The amount that Varun had initially is 3300 + 100
k 2 1 / 12 = Rs. 3400 Choice (B)
c2 = 27 × 12
42. As PQ = 840, RP = 420 R
c = 9 × 2 = 18
4 and QP = 420 3 420
∴9a + 5c = 9 × + 5 × 18 = 94 Choice (A) 60°
9 Time of arrival of the
train at Q P
38. Given 3a + 5b + 7c = 1.25 k → (1) 420 3 840
2a + b + 3c = 0.75 k → (2) = hours + 9 a.m.
84 3
(1) × 2 – (2) × 3 gives
= 2 p.m. 420 3
6a + 10b + 14 c = 2.5 k he latest time by which 30°
6a + 3b + 9c = 2.25 k Raju has to arrive at Q
= 1:35 p.m.
7b + 5c = 0.25 k The time he takes to Q
1 840
∴ 7b + 5c = k cover RQ =
210
4
∴7b + 5c is 25% of k Choice (A) 3
5
= 4 3 hours = 6 hr 56 min. While Arun covers AB, the bike covers AB + BC + CB.
AB AB + BC + CB
The latest time at which Raju can leave ∴ =
R = 1:35 p.m. – 6 hr 56 min = 6:39 a.m. ≈ 6:40 a.m. 5 45
Choice (2) ⇒ 9AB =(AB + BC + CB) -------(1)
While Tarun covers CD, the bike covers CB + BC + CD.
Solutions for questions 43 and 44: ∴ 9 CD = CB + BC + CD ------- (2)
Consider Arun and Tarun.
Let the cruising speed of the plane be S kmph. Each covers part of the distance at 5 km/hr and the rest
Let the time difference between P and Q be t hours. of 45 km/hr. Both take the same time to cover AD.
Time taken by the plane to travel from P to Q ∴ Each has to cover the same distance at the lower
= 5:00 p.m. – t hours – 7:00 a.m. speed (and hence the same at the higher speed) i.e.,
= (10 – t) hours. AB = CD.
Similarly time taken by plane to travel from Q to P Also BC + CB = 8 AB (from 1) or BC = 4 AB.
= (10 + t) hours. Alternately, (1) and (2) ⇒ AB = CD.
Effective speed of a plane traveling from P to Q ∴ ab : bc : cd = 1 : 4 : 1
= (S + 100) kmph. AB = 6, BC = 24, CD = 6
Effective speed of a plane traveling from Q to P The time taken (in hours) is (considering Arun)
= (S – 100) kmph 6 30  1 1
4800 + = 6  + 
10 – t = 5 45  5 9 
S + 100
6 (14 ) 28 13
4800 = = =1 Choice (D)
10 + t = 45 15 15
s − 100
Solving for S and t,
48. Let the numbers (in ascending order) be L, L + 1, L + 2.
S = 500 and t = 2
Given 3[L3 + (L + 1)2 + (L + 2)] = (3L + 3)2
⇒ 3(L + L + 3L + 3) = 9(L + 2L + 1)
3 2 2
43. Required time = 5:00 p.m. – t hours
⇒ 3L – 6L – 9L = 0 ⇒ L =) or L – 2L – 3 = 0
3 2 2
= 3:00 p.m. Choice (2)
⇒ L(L + 1) (L – 3) = 0 ⇒ L = 0, –1 or 3.
44. 500 km/hr Choice (3) As L > 0, L = 3 Choice (1)
3
Solutions for questions 45 to 49: 49. For any cubic equation in x, in which the coefficient of x
is 1, the coefficient of x is equal to the sum of the product
45. After B travels for 48 min, (and covers 24 km) let us say of the roots taken two at a time.
A reaches Q and B reaches R. ∴q = ab + bc + ca. As a, b and c are integers,
P q must be an integer.
Q
Let us go by the choices. Choice (5) is the least among
the choices but is not an integer.
24 ∴it is ruled out. Choice (4) is the second least.
74 If q = –1, ab + bc + ca = –1.
This is possible if a = –1, b = 0 and c = 1 and if
R a = 1, b = 0 and c = –1.
∴The least possible value of q = –1. Choice (4)
As QR = 2 (37), PR = 2 (12)
PQ = 2 372 − 122 = 2 (5) (7) = 70 Solutions for questions 50 and 51:
70 4 is a root of g(x) = 0. Let the other root be y.
To cover 70 km, A must have travelled for hrs or
45 g(x) = c[(x – 4) (x – y)] where c is a constant
5 1 1 = c[x2 – (4 + y)x + 4y].
1 hr. = 1 hr 33 min = 93 min. Also 8g(3) = – 3g(6)
9 3 3
To cover 24 km, B has travelled for 48 min. ∴ 8[c(9 – (4 + y) (3) + 4y)]
= − 3c[36 − (4 + y)6 + 4y]
1 1
∴ B started (93 – 48) min after A or 45 min after As c ≠ 0, − 24 + 8y = − 36 + 6y ⇒ y = –6
3 3 ∴g(x) = c[x2 – 2x – 24]
9:00 a.m. No information is available to find c.
∴ B started at 9:45:20 a.m. Choice (B) ∴p, q and r cannot be found
46. The distances that A, B, C run by the time A and B 50. The other root of g(x) = 0 is – 6. Choice (3)
finish the race are tabulated below:
A B C 51. p + q + r cannot be found. Choice (5)
By the time A finishes x x – 180 x – 351
By the time B finishes x x – 198 Solutions for questions 52 and 53:
x − 180 x − 351
∴ = The data and calculations are tabulated below.
x x − 198
⇒ x – 378x + 180 (198) = x – 351x
2 2
No. Cost Cost
⇒ 27x = 180 (198) x 200+ px + qx2
⇒ x = 20 (66) = 1320 Choice (D) 10 200 + 10p + 100q 9a(say) → (1)
47. Varun’s bike travels at 45 km/hr. Arun and Tarun walk 15 200 + 15p + 225q 17a → (2)
at 5 km/hr. Arun covers AB on foot. Tarun covers CD on 20 200 + 20p + 400q 28a → (3)
foot.
(2) – (1) ⇒ 5p + 125q = 8a
• • • • (3) – (2) ⇒ 5p + 175q = 11a
A B C D
6
58. Let α, β are the roots of the equation x – (k + 7) x –
2
3a a
∴q= and p = (3k – 15) = 0
50 10
∴α +β = k + 7; αβ = 15 – 3k
 a   3a 
(1) ⇒ 200 + 10   + 100   = 9a ⇒ a = 100 (α – β)2 = (α+β)2 – 4αβ
 10   50  = (k + 7)2 – 4 (15 – 3k)
⇒ p = 10, q = 6 = k2 + 14 k + 49 – 60 + 12 k
∴ The cost of production of x items is 200 + 10x + 6x2 = k2 + 26 k – 11
⇒ (k + 13) – 11 – 169
2
The selling price is 100x.
∴ The profit is p = –6x2 + 96x –200. (α – β)2 = (k – 13)2 – 180
When a < 0, the expression ax2 + bx + c has a maximum ∴ The minimum value of (α – β)2 is –180.
−b Choice (B)
value and it occurs when x = .
2a
59. The data is tabulated below:
96
∴ The profit is maximum when x = = 8, and the
12
Cost of each Number of cups
maximum profit is –6(64) + 96(8) – 200 = 184. Total Cost
cup of tea
52. For the profit to be maximum, the daily production has x n 90
to be 8. Choice (3)
x + 1.50 n – 10 90
53. The maximum profit is Rs.184. Choice (5)
To get rid of the fraction, we can multiply by 2.
Solutions for questions 54 to 61:

54. E(y) = ay2 + by + c Cost of each cup Number of cups


Total Cost
Where a, b and c are constants. (Hypothetically) of tea
E(y) is minimum when y = –3.
2x n 180
−b
∴ = –3 2x + 3 n – 10 180
2a
i.e., b = 6a
E(–3) = 1 We see from the options that only integral values of 2x
⇒ 1 = 9a – 3b + c have to be considered. 180 = 22 (32) (5). There are 3 (3)
⇒ 1 = –9a + c → (1) (2) or 18 factors or 9 pairs of factors. They are
E(–2) = 2 ⇒ 2 = 4a – 2b + c 1 (180) 5 (36)
⇒ 2 = –8a + c → (2) 2 (90) 6 (30)
Solving (1) and (2), a = 1 3 (60) 9 (20)
And c = 10. 4 (45) 10 (18) and 12 (15)
∴ b = 6 and hence E(y) = y2 + 6y + 10.
We need to look for two numbers that differ by 3 (in the
∴ E(4) = 50. Choice (3)
left column) and the corresponding numbers in the right
55. Since when x = 1; both the equations become a + b + c = 0 column should differ by 10.
1 is the common root of the equations 2x = 6 and n = 30
⇒a+b+c=0 follows (∵ 2x + 3 = 9 and n – 10 = 20)
⇒ a + b + c = 3abc
3 3 3
Choice (B) ∴ The price is Rs.3 per cup. Choice (A)

56. We know that if α, β, γ are the roots equation 60. Let the number of common terms be N. The first AP has
x3 + p1x2 + p2x + p3 = 0; then α + β + γ = –p1 a common difference of 5, while the second has a
αβ + βγ + γα = p2 common difference of 4.
α β γ = – p3 ∴ the sequence of common terms will be in AP whose
∴Given α, β, γ are the roots of the equation, common difference is LCM. (5, 4) i.e., 20.
x3 + 2x3 – 5x – 6 = 0 Let there be N terms common to the given sequences,
∴α + β + γ = –2; where the Nth term = 29 + (N – 1) (20)
αβ + β γ + γα = –5 = 9 + 20N. This cannot exceed min (474, 485) or 474
αβγ=6 i.e.9 + 20N ≤ 474
Now given αβ, βγ, γα are the roots of the equation N ≤ 23.25
x3 + px2 + qx + r = 0; ∴N must be the greatest natural number ≤ 23.25 i.e.,
∴αβ +βγ + γα = –p = –5 = –p
23. Choice (5)
⇒p=5 Choice (C)
61. Suppose the prices of the two varieties are equal on
57. Given α – β = 9 day N.
|α| – |β| = 5; Suppose n ≤ 200.
Squaring on the both sides Then 110 + 0.2n = 97 + 0.25n
α2 + β2 – 2 |α β| = 25 → (1) n = 260 which is not possible.
⇒ α – β = 9 ⇒ α +β – 2αβ = 81
2 2
∴ n > 200
⇒ α2 +β2 = 81 + 2αβ → (2) If n = 200, price of Nilgiris coffee = Rs.150.
Substitute the value of α2 +β2 in (1) ∴ 97 + 0.25n = 150 ⇒ n = 212
⇒ 81 + 2 αβ – 2 |αβ| = 25 2006 is not a leap year. ∴ February 2006 has 28 days.
81 – 25 = 2 |αβ| – 2 αβ 212 = 31 + 28 + 31 + 30 + 31 + 30 + 31. ∴ on July 31st,
56 = –2 αβ – 2αβ (∵αβ is negative) prices of the two varieties of coffee would be equal.
56 = –4αβ Choice (3)
∴αβ = – 14 Choice (A)
7
Solutions for questions 62 and 63:  1 2 1 
= 15  − + 
When x is even, we have to multiply the previous term in the  1000 1111 1224 
other sequence by M to get the xth term in either g or h. 1429
When x is odd (and ≥ 3) we have to multiply the previous = Choice (C)
5666100
term in the other sequence by N to get the xth term in either
g or h.
65. The least five digit number that when divided by
∴ We will find it convenient to tabulate the terms of g and h 8 leaves a remainder of 4 is = 10004 and the greatest
in 2 columns as shown below. such five digit number is = 99996
g H ∴The number of five digit numbers which when divided
M N by 8 leave a remainder 4 is 11250
MN M2 Their sum is =
11250
[10004 + 99996] = 618750000
M2N M2N 2
MN2 2
M3N Similarly the sum of five digit number which leave a
M3N2 M2N3 11250
remainder 6 when divided by 8 is [10006 + 99998]
M3N3 M4N2 2
MN4 3
M3N4 = 618772500
Total sum = 1237522500. Choice (D)
62. g2 + h2 = M(N + M)
66. Let a and d be the first term and common difference of
g4 + h4 = M2N(N+M)
the progression
g6+h6 = M3N2(N+M)
x
Given sn = 2400
−1 s2n – sn = 7200
We have gx + hx = M(MN) 2 (M + N)
∴s2n = 9600
∴ choice (3) follows. Choice (3)
n
⇒ [2a + (n – 1)d] = 2400 → (1)
63. g1 + h1 = 1 2
3 n[2a + (2n – 1)d] = 9600 → (2)
g3 + h3 = MN(M + N) = = 0.1875 ⇒ (1) ÷ (2) we get
n[2a + (n − 1)d] 4800
16
g5 = M N and h5 = M N ∴ g5 + h5 = M2N2(M + N)
3 2 2 3
=
n[2a + (2n − 1)d] 9600
9 7.68
= > = 0.03 4a + 2nd – 2d = 2a + 2nd – d
256 256
2a = d
g6 = M N and h6 = M4N2
3 3
a 1
g7 = M4N3 and h7 = M3N4 ⇒ = Choice (C)
∴ g7 + h7 = M3N3(M + N) d 2
27 40.96 67. Let t1 = a; t3 = b
= < = 0.01
4096 4096 a+b b−a −(a + b)
∴ 7 is the least odd value of x satisfying gx + hx < 0.02. then t2 = , t4 = , t5 = –a, t6 , t7 = –b;
Choice (3) 2 2 2
a−b
t8 =
Solutions for questions 64 to 76: 2
90 Again t9 = a;
1
64. Let the series be ∑ After every 8 times again first term be repeated.
x = 1 ( x + 9 ) ( x + 10 ) ( x + 11) ( x + 12) ∴ t31 = t7 = –b
1 t27 = t3 b
Consider f (x) =
( x + 9) ( x + 10 ) ( x + 11) ( x + 12) t −b
∴ 31 = = −1 Choice (C)
t 27 b
1 1 1  1 1 1 
=  −  –  − 
6  x + 9 x + 12  2  x + 10 x + 11 st rd th
68. The 1 , 3 , 5 terms form a GP and the rest form an
90 1 1 1  1 1 1  AP
∴ ∑  − −  −  2 + 4 + 8 + …. + 220 = 2(220 – 1)(G.P)
x =1 6  x + 9 x + 12  2  x + 10 x + 11
and 3 + 6 + 9 + ……+ 60 = 630(A.P)
= ∴ The required sum is 221 – 2 + 630 = 221 + 628
1 1 1 1 1 1 1 1 1 1 1 1 1  Choice (B)
− + − + − + − + − + ... + −
6 10 13 11 14 12 15 13 16 14 17 99 102 
 3  G(3 )
69. G(3) = G   =
 1  G(1)
1 1 1 1 1 1 1 
– − + − + ... + −
2  11 12 12 13 100 101
∴G(1) = 1
1 1 1 1 1 1 1  11 1   
= + + − − − − −  
6  10 11 12 100 101 102  2  11 101 3
G(9) = G  
1
1 1 1 1 1 1 1  1  101 − 11  
=  − + − + − −   3
6 10 100 11 101 12 102 2 101 × 11
2
 1
1 9 90 90  1  90   
=  + +  −   G(3 ) G(3 ) (G(3))
2
=  =
9 1
6  100 101 × 11 12 . 102  2  101 × 11 = = =
 1   G(1)  G(1) 1 81
G   
=
90  1
 +
1
+
1

3 
   
3 G(3 ) 
6  1000 1111 1224 1111 Choice (1)
8
70. G (1) = 5400 ∴The 150th term of 5, 12, 19, 26…is t75 = a + 74d
= 5 + 74 × 7 = 5 + 518 = 523 ∴f(150) = 523
G(1) + G(2) = ∑ G(y ) = 2 G(2)
2 2

y =1
The 75th term of the series of 2, 9, 16, 23….
T38 = 2 + 37d
G(1) + G(2) = 4G(2) = G(2) = 2 + 37 × 7 = 2 + 259 = 261
Similarly G(3) = 900, f(75) = 261
G(4) = 540, G(5) = 360 and
∴f(150) : f(75) = 523 : 261 Choice (A)
1800
G(6) = Choice (3)
7 74. Sum of the ages of the members twelve years ago
= 250 years.
10 2 Four years later, the sum of their ages = 290 years.
71. Given f(xy) = f(x) + f(y) + f(x) + f(y) – and f(6) =
9 3 That year, a member aged 50 years died and a child
10 was born.
f(6.1) = f(6) + f(1) + f(6) + f(1) – Sum of the ages of the members after the child’s birth
9
= 240 years.
10 Four years later, a member aged 50 years died and
= f(1) [1 + f(6)] =
9 another child was born.
 2 10 Sum of the ages of the members after this birth = 230 years.
⇒ f(1) 1 +  = Sum of the present ages of the members = 270 years.
 3 9 Present average age of the members = 27 years.
5 10 Choice (2)
⇒ f(1) =
3 9
75. The number, the average age and the combined age of
2 the two groups are tabulated below.
f(1) =
3 Group Number Av.Age Combined Age
 1  1  1 10 A m x+6 mx + 6m
f   = f   + f(1) + f   f(1) – B m+3 x mx + 3x
6 6 6 9 The difference of the combined ages is 3 (x – 2m)
  1  10 (The total age of B is more and it will continue to
f(1) 1 + f   = exceed that of A). After 4 years, it would be 12 more
  6  9
than this (as B has 3 more children). If 12 more than a
  1  2 10 number is double the number, the number is 12, i.e.,
1 + f   = x – 2m = 4. The possible values for m, (the number of
  6   3 9
children in A), 2m + 3 (the total number of children) and
 1 5 x (the average age of B) are tabulated below:
1 + f   =
6 3 M 2m + 3 x
1 5 6
 1 2
f   = Choice (A) 2 7 8
6 3 3 9 10 Choice (B)
x 76. The negative deviation is 10 M + 5 (M + 2)
72. Given f(x) =
1+ x The positive deviation is 10 (7)
∴ 10 M + 5 (M + 2) = 7 (10) ⇒ M = 4
 x  The total number of students is 4 + 6 + 6 + 7 = 23.
f(f(x)) = f  
 1+ x  Choice (D)
x Solutions for questions 77 and 78:
1 +x x
= =
x 1 + 2x The ratio of the amount bet on Q, R, T is 1 : 2 : 3. There was
1+
1+ x no gain and no loss. We have the following possibilities.
The first table shows the bets amounts on the 3 horses and
x
the different ways in which Mohan could have got back the
1+ x x
f(f(f(x)) = = total amount he bet.
1+ 2
x 1 + 3x The second table shows the corresponding positions of the
1+ x horses. ‘−’ indicates 4th position or 5th position.
x Positions
similarly f(fn times ) = Q R T
1+ nx Q R T
x Bet 1 2 3
∴ f(f(f(f(f(f(x))))))) = Return
1+ 6 x 0 3 0 − 2 −
factor
3 3 3 0 1 2 − 3
= f(f(f(f(f(f(3))))))= = Choice (C)
1 + 6.3 19 4 1 0 1 3 −

73. f(x + 2) = f(x) + 7 77. We consider the options.


∴ f(x + 2) – f(x) = 7 (1) At least 2 horses finished before P, i.e. p = 3, 4 or
put x = 1, f(3) = f(1) + 7 = 9 5 possible.
put x = 2, f(4) = f(2) + 7 = 12 (2) There were 3 horses between Q and P. Possible
put x = 3; f(5) = f(3) + 7 = 16 (Q, P) could be (1, 5) or (5, 1)
i.e f(1), f(2), f(3), f(4), f(5)….are as follows (3) T finished last is possible
2, 5, 9, 12, 16, 19, 23, 26, 30…..i.e first term, third, fifth (4) S came second is possible
term are in A.P with common difference 7 and second (5) There were 3 horses between T and R. Not
term, fourth term, sixth term,… are A.P with common possible i.e. {T, R} cannot be {1, 5} or {5, 1}
difference 7 Choice (5)
9
78. Using the above conclusions and the statement that S Section B-Sohan A-Sohan Mohan, Sohan
came fourth we get the following cases.
Av. Height 140 140 140
P Q R S T
(1) – 2 4 – No. 39 39 2
(2) 2 – 4 3
(3) 1 3 4 –
As the average height of B (without Sohan) is equal to
Where ‘–’ indicates 4 or 5 (As s = 4, ‘–’ means 5) the average height of (A and Sohan), each is equal to
We see that (1) is not possible as {Q, T} = {4, 5} 140. As the average height of the 39 students of A
∴ S cannot be 4. (except for Mohan) is equal to the average height of the
Statement (5) cannot be true. Choice (5) 39 students of B (except for Sohan) this average is also
140.
Solutions for questions 79 and 80: ∴ If we combine the two statements, we can conclude
that Mohan’s height is (140 + 40)cm and Sohan’s height
79. From statement A, there were 169 players. Therefore, a is (140 – 40) cm. Choice (3)
player gets a bye in the first round. This may or may not
be the champion. Statement A is not enough. 82. Using statement A, volume of the material used to
36000πkg
From statement B, we know that the champion got only make the tank = = 9π m3
4
1 bye. But we do not know the total number of players. kg / cc
1000
By combining the statements, we get the following 4
table. Volume of water the tank can hold = π(63) – 9π
3
Round 1 2 3 4 5 6 7 8 = 23 79πm3 = 279π kilolitres.
∴ the tank will meet the company’s requirements.
No. of
84 42 21 11 5 3 1 1 A is sufficient.
Matches
Using statement B, if the tank’s inner radius is 4.5 m,
Player getting
1 1 1 0 1 0 1 0 4
bye volume of water it can hold = π (4.5)3
3
As the champion got only 1 bye, he must have played = 121.5 π m3 = 121.5π kilolitres
8 − 1 or 7 matchers. Choice (4) If its inner radius is close to 6 m, it can hold
approximately 288π kilolitres of water.
80. In round 1, there were N players, where 129 < M < 256.
∴ we cannot be sure of whether the tank will meet the
From statement B, 1 player received a bye from round 3
company requirements. B is not sufficient.
to round 4.
Choice (1)
This is not enough to determine M. (There could have
been byes in earlier rounds.)
83. Using statement A,
From statement B, only one player received a bye. But
PQ + QR + PR = P2 + R(2P)
this could have been in round 1 (say of M = 131) or
round 2(say of M = 130) etc., We do not know the values of P, Q and R.
∴ We cannot find the maximum value.
By combining the statements, we get the following A is not sufficient.
table. Using statement B,
(P + Q + R)2
Round 1 2 3 4 5 6 7 8
= P2 + Q2 + R2 + 2PQ + 2QR + 2PR → (1)
Paired player 252 126 62 32 16 8 4 2 (P – Q)2 + (Q – R)2 + (P – R)2 ≥ 0
Player getting ∴ P2 + Q2 + R2 ≥ (PQ + QR + PR)
1 (1) ⇒ (P + Q + R)2 ≥ 3(PQ + QR + PR)
bye
∴ maximum value of 3(PQ + QR + PR)
We can fill up this table (from right to left) and = (P + Q + R)2 = 842.
[(
determine that N = 252 26 − 1 22 ) ] Choice (4) ∴ maximum value of PQ + QR + PR can be found.
B is sufficient. Choice (2)
Solutions for questions 81 to 84:
84. Using statement A, we do not know anything about the
81. The data is tabulated below. locations of A, B and D with respect to C or P.
Section B BA A ∴ We cannot find why he failed.
Av. Height b 140 a A is not sufficient. P B
A •
No. 40 80 40 Using statement B,
If Mohan goes to B and Sohan to A, the average height PD
PC = 3K K
of the two sections would interchange. This means the
3
average height of the 39 other students in B (other than
Sohan) is equal to the average height of the 39 other Let PD be 3k
student in A (other than Mohan). 3k
So PC = =k D C
From (A), b–a = 2. ∴ b = 141 and a = 139. (As the 3
number of students in the two sections is equal, the Let x be the side of the square.
deviations of the average height from the average of the We known that PC or PD must lie between x and 2 x,
entire class are equal). In B, if we replace Sohan with
Mohan, the average height becomes 141, i.e. Mohan is since PD (or PC) ≤ Diagonal of the square.
80 cm taller than Sohan. But we cannot get the height of So PD/PC lies between 1 / 2 and 2 . It can never
either. be 3. So it is not possible, thus he failed.
Hence statement B alone is sufficient.
From (B), we can tabulate the data as shown below.
Choice (2)
10
Solutions for questions 85 and 86: 90. Let the cost price of each mango for Ashok be Rs.x.
2040
Suppose Mohan invested Rs.x in alternative 1 and Rs.y in ∴ the selling price of each mango = Rs.
96
alternative 2.
= Rs.21.25.
If there was a rise in the market his return (say R1)
21.25
 −8 5  As loss is 15%, CP per mango = = 25.
=  x + y 0.85
 100 100 
If he makes 30% gain, then the SP of each mango
If there was a fall in the market his return (Say R2)
 10 4  =
1.30
(25) = Rs.32.50
=  x − y 100
 100 100  2600
∴ Required number = = 80. Choice (A)
5y 32.50
R1 has its maximum value of when x = 0 and it
100
decreases as x increases. R2 has its minimum value of 91. Let the monthly incomes of A, B and C be Rs. 6x, Rs.7x
4y and Rs.8x respectively.
− when x = 0 and it increases as x increases. Let the monthly expenditures of A, B and C be Rs.5y,
100
Rs.6y and Rs.10y respectively.
The assured return (i.e. the smaller of R1 and R2) has its Monthly savings of A, B and C are Rs.(6x – 5y),
maximum value when R1 = R2, i.e. when Rs.(7x – 6y) and Rs.(8x – 10y) respectively.
−8 x 5y 10 x 4
+ =
100 100 100 100
− y 8x – 10y =
37.5
(8x ) = 3 (8x ) = 3x
100 8
⇒ y = 2x 5x = 10y
2x x = 2y.
His return from either alternative = Rs. .
100 ∴ 6x – 5y = 7y and 7x – 6y = 8y.
His total investment = Rs.3x. 7y
Required % = (100)% = 87.5%. Choice (D)
2 8y
∴ his return percentage = %
3
92. Let the sum be Rs.P.
2 2
85. The greatest assured return for Mohan is %.  16 
3 Difference = P   = 384.
Choice (3)  100 
384
86. y = 2x gives the maximum assured return percentage of P= .10000 = 15000. Choice (B)
256
2 1 1
%. ∴ = .
3 2 2 93. Let the loan amount be Rs. P. Let the rate of interest be
Choice (3) follows. Choice (3) R% p.a.
9PR
Solutions for questions 87 to 140: P+ = 90,000 → (1) and
100
12PR
87. Let the initial price of coffee be Rs. 10 per kg. Let the P+ = 105,000 → (2)
initial consumption be 10 kg. Initial expenditure = Rs.100. 100
New price = 10(1.4) = 14. 3PR
New expenditure = 100(1.12) = 112 (2) – (1) ⇒ = 15,000
100
112
New consumption = = 8. PR
14 ∴ = 5000
100
∴ Percentage by which Raj must decrease his
4PR
10 − 8 Required amount = P +
consumption = × 100 = 20%. Choice (C) 100
10
= 90000 – 4(5000) = Rs.65000. Choice (B)
88. Let the populations of X, Y and Z in 2008 be 300, 500
and 600 respectively. 94. Let the required time be x minutes.
Populations of X, Y and Z in 2009 were 300(1.1) = 330; Time taken to fill the tank = 50 minutes.
500(1.125) = 562.50 and 600(1.15) = 690 respectively. 1 1 1
− =
Total population in 2008 = 1400 x 200 50
Total population in 2008 = 1582.50 ⇒ x = 40. Choice (C)
182.50
Percentage increase = × 100 ≅ 13%.
1400 95.
Choice (C) Total
Number of Number Wall length
Men (M) number of
89. Let the cost of each ball pen be Rs.x. hrs 1 day of days (in m) (W)
hours (H)
3600 3600
− = 90. 8 8 8 64 8
 20  x
x 1 − 
 100  16 ? (say x) 16 16 16
 5 1
⇒ 3600  −  = 90 M1H1 M2H2
=
 4x x  W1 W2
900
⇒ = 90 ⇒
8 . 64 16 . 16 . x
=
x 8 16
⇒ x = 10. Choice (B) ⇒x=4 Choice (D)
11
96. As the total man hours required to complete the job is 100. Let Q Q1 and P P1 be the towers T1 and T2. It is given
constant, we have that ∠QRQ1, and ∠QSQ1 are both 'α' where R and S
1(50)(20)(9) = 1(50)(x)(9) + 1(45)(12.5)(8) are the two points on the ground. Again P1 Q = R P1
9000 = 450x + 4500 = SP1 = 30 feet, where R, P1 and S are collinear.
⇒ x = 10. Choice (D)
Q

97.
T (30%) C (40%) •
P • Q1
α α
a% •
R 30 P1 S

b% Again R and S are equidistant from Q [ Q ∠QRQ1


= ∠QSQ1 = α] Thus QR = QS
Let the strength of the class be T. Q
Number of students who like both tea and coffee
a 20  30 
= (T ) =  T
100 100  100  30
⇒ a = 6.
Required percentage = b% = 100% – (Percentage of
students who like either tea or coffee) = 100% – (30% +
40% – 6%) = 36%. Choice (D) R P1 S
30 30
98. In the above ∆ RQS is isosceles.
Son Thus ∠QP1S = ∠QP1R = 90°
Star Now, in the ∆ QRS, QP1 is the median to RS which is
180 a 210 1/2 RS. This is possible only when ∠RQS = 90°. Thus
90
c b the area of ∆ QRS = 1/2 (RS) (QP1) = 1/2 (60) (30)
= 900 ft2. Choice (D)
150
101. P
Zee

a+ 90 + b + 210 ≤ 340 [since at most 340 residents 10m


watch Star Plus]
∴ a + b ≤ 40 – (1)
180 + 90 + a +b + c + 210 = 540 [ Q 540 residents watch 45 60 60
at least one of Sony TV and Star Plus]
∴ a + b + c = 60. R S Q T
(1) ⇒ c ≥ 20.
PQ.
Required number = Minimum value of c = 20. = sin 45
Choice (C) PR
1
99. Let the number of boys and the number of girls be PQ = 10. = 5 2m.
b and g respectively. 2
Boys Girls Required distance = ST
Number of persons liking cricket 0.6 b 0.5 g = SQ + QT.
Number of persons liking volleyball 0.45 b 0.55 g PQ PQ 2 PQ 2(5 2 ) 10 6
0.6 b + 0.5 g = 7 + 0.45 b + 0.55 g = + = = = m.
tan 60 0 tan 60 0 3 3 3
0.15 b = 7 + 0.05 g i.e., 3b = 140 + g – (1)
Choice (B)
0 .6 b − 0 .5 g 16
= .
0.45 b − 0.55 g 5 102 . If p, q, r are any 3 real numbers, pq + qr + rp ≤ p2 + q2 + r2.
If p, q, r satisfy the condition that the sum of any two
± (0.6 b − 0.5 g) 16 exceeds the third (or equivalently the sum of the two
= .
(0.45 b − 0.55 g) 5 smaller numbers exceeds the greatest), then all of
± (3 b – 2.5 g) = 7.2 b – 8.8 g. them have to be positive.
3b – 2.5 g = 7.2 b – 8.8 g or –3b + 2.5 g = 7.2 b – 8.8 g p + q > r ⇒ pr + qr > r2.
6.3 g = 4.2 b or 11.3 g = 10.2 b q + r > p ⇒ pq + pr > p2.
b 3 b 113 p + r > q ⇒ pq + qr > q2.
i.e., = or = . Adding these and simplifying, we get
g 2 g 102
p2 + q2 + r2 ≤ 2 (pq + qr + rp)
b 3
If = , (1) ⇒ b = 60 and g = 40 1 pq + qr + rp
g 2 ∴ < 2
2 p + q2 + r 2
b 113
If = , (1) ⇒ then 60% of b and 55% of g are not 1 1
g 102 i.e., < cos θ and hence < cos θ ≤ 1
2 2
integers.
∴ b = 60 and g = 40. Hence, the required strength i.e., 0° ≤ θ < 60°.
= 100. Choice (B) θ can’t be 80° Choice (D)
12
103. As the LHS consists of power of 3’s, RHS should also = 982 – 2 = 9602
be expressed in the form of power of 3. We need to a4 + b4 9602 4801
list the powers of 3 until we get numbers greater than ∴ = = Choice (D)
118098. a3 + b3 970 485

3 243 19683 107. The two bases in the logs are 2 and 4. Let x = 22p = 4P
9 729 59049 Applying log on both sides
27 2187 177147 ∴ log4x = P and log2 log4x = log2P.
81 6561 log2x = log24p = log222p = 2p.
We can check that 177147 – 59049 is 118090. log 2 2P
log4log2x = log42P =
∴ 33x + 2 – 92x – 1 = 311 – 95. 2
If 3x + 2 = 11 (as x = 3) then 2x – 1 = 5. log 2 2p
∴ x = 3 and 6x + x6 = 63 + 36 = 216 + 729 = 945. Given log2P =
2
Alternative Method: ⇒ 2 log P = log 2 + log P
118098 can be expressed as ⇒ P = 2 ⇒ x = 2 = 16.
4

= 9 × 13122 = 9 × 9 × 1458
= 92 × 9 × 162 Alternative Method:
118098 = 93 × 9 × 18 Substitutes the choices, only Choice (1) satisfies.
= 95 × 2 Choice (A)
= 310 × 2 x x + 1
108. As log77, log7(4 + 5), log7 (4 – 1) are in AP, the
= 310[3 – 1] = 311 – 310 Choice (A)
3 numbers are in GP, i.e.,
x+1 x
log77 + log7(4 – 1) = 2log7(4 + 5)
104. Consider the following expression:
= log7 7(4x + 1 – 1) = log7(4x + 5)2
1 1
( )
= 7 (4x + 1 – 1) = (4x + 5)2
(n + 1) n + n n + 1 n(n + 1) n + 1 + n Let 4x = y.
28 y – 7 = y2 + 10y + 25
n +1 − n 1 1 ⇒ y2 – 18y + 32 = 0
= = −
n(n + 1) n n +1 ⇒ y = 16 or y = 2, i.e., 16 = 4x or 2 = 4x
⇒ x = 2 or x = ½ Choice (C)
1 1 1
∴ = −
17 16 + 16 17 16 17 109. Given (x2 + 4x – 32) (x2 + 2x – 8) > 0
1 1 1 (x + 8) (x – 4) (x + 4) (x – 2) > 0
= − The critical points are –8, –4, 2, 4.
18 17 + 17 18 17 18
Ι ΙΙ ΙΙΙ ΙV V
⋮ ⋮ ⋮
–8 –4 2 4
⋮ ⋮ ⋮
1 1 1 When x = 0; the equation is true.
= − The values of x in first, third and fifth region are
121 120 + 120 121 120 121 satisfied.
1 1 1 1 ∴ The values in second, fourth region are not satisfied.
∴ The required sum is − + – The number of values of x that the inequation do not
16 17 17 18
satisfy are x ∈ [– 8, –4] ∪ [2, 4] ie 8 values.
1 1 Choice (B)
+.... –
120 121
a2 b2
=
1

1
=
1

1
=
7
Choice (B) 110. Given − ≤ ( a – b)
4 11 44 b a
16 121
a3 − b3
– (a –b) ≤ 0
1 ab
105. x+ = 3 → (1)
x (a 3 − b 3 ) − (a − b)(ab)
≤0
Squaring on both sides, we get ab
1
∴ x2 + = 7 → (2) (a − b)(a 2 + ab + b 2 ) − ab(a − b)
x2 ≤0
ab
1
Combing both sides, we get x6 + + 3x2 a−b
x6 ⇒ ≤ 0 (∵ a2 + b2 is always positive)
ab
1  2 1  ⇒ (a – b) ab ≤ 0
 x +  = 73
x2  x2  ⇒ (a – b) ≥ 0 and ab < 0 (∵ as ab ≠ 0) or (a – b) ≤ 0
1 and ab > 0
x6 + 6 + 3(7) = 343
x ⇒ a ≥ b and ab < 0 or a ≤ b and ab > 0. Choice (C)
6 1
x + 6 = 322 Choice (C) 4x 2 − 9x + 8
x 111. > 3.
x 2 − 5x − 6
106. a = 5 – 2 6 , b = 5 + 2 6 4 x 2 − 9 x + 8 − 3( x 2 − 5 x − 6)
>0
∴ a + b = 10 and ab = 1 x 2 − 5x − 6
a3 +b3 = (a + b)3 – 3ab(a + b) = 103 – 3(10) = 970
( x 2 + 6 x + 26)( x 2 − 5 x − 6)
a4 +b4 = (a2.+.b2)2 – 2a2b2 = [(a + b)2 – 2ab]2 – 2a2b2 >0
(102 – 2)2 – 2 ( x 2 − 5 x − 6)2
13
as x2 + 6x + 26 is always positive 1 1 9 3
⇒ x – 5x – 6 > 0
2
118. T1 = 1+ + = =
(x – 6) (x + 1) > 0 12 22 4 2
∴ x < –1 or x > 6 ∴ s1 = t1 =
3
=2–
1
⇒ (–∞, –1) ∪ (6, ∞). Choice (D) 2 2
1 1 1 1 36 + 9 + 4 7
112. Given x1 x2 x3 x4 x5 x6 = 1 t2 1+ + = 1+ + = =
If (x1 + 4) (x2 + 4) (x3 + 4) (x4 + 4) (x5 + 4) (x6 + 4) is 22 32 4 9 36 6
minimum when x1 = x2 = x3 = x4 = x5 = x6 = 1. 8 1
∴ s2 = t1 + t2 = =3–
∴The minimum value is 5 × 5 × 5 × 5 × 5 × 5 = 15625. 3 3
Choice (A) 1 1 144 + 16 + 9 13
t3 = 1+ + = =
113. a ⊕ b = 4a + 3b + 7ab 32 42 3242 12
a ⊕ a= 4a + 3a + 7a2 32 13 15 1
∴ s3 = + = =4–
= 7a(1 + a) 12 12 4 4
b ⊕ b = 4b + 3b + 7b2 1
= 7b (1 + b) Similarly s2005 = 2006 – Choice (2)
2006
Given a ⊕ a < b ⊕ b
7a (1 + a) < 7b (1 + b)
2 2 119. We can arrange the elements of A in 2 columns
a+a –b–b <0 (1 and 2) as shown below. The numbers that we get
(a – b) + (a2 – b2) < 0 by increasing each odd number by 3 and increasing
(a – b) × (a + b + 1) < 0. Choice (A) each even number by 1 are shown in the column 3
and 4 respectively.
 3  3 1 2 3 4
114. Given x @ y = y 4 x +  + x 4 y +  3 4 6 5
 x   y 5 6 8 7
3y 3x x y y x 2N + 1 2N + 2 2N + 4 2N + 3
= 4xy + + 4xy + = 8xy +  +   + .
x y y x  x y
   P is the average of the resulting odd numbers (i.e.
  column 4). Q is the average of the resulting even
 
1 1  6 1  1  numbers (i.e. column 3). We see that P–Q = –1.
6@ = 8.6 +3 + =8+3  36 + 
6 6  1 6   36  Choice (3)
 
6 6
1 1 3
3891 4179 120. = −
=8+ = x 18 y
36 36
1 3 1
Choice (C) + =
x y 18
115. On replacing two numbers say p and q by p + q – 1, 18(y + 3x) = xy
the following are the consequences. xy – 18y – 54x + (18) (54) = (18) (54)
(i) the number of numbers on the board decreases by 1. (x – 18) (y – 54) = 35 × 22
(ii) the sum of the numbers on the board decreases by 1. x is odd and less than 100.
The above consequences mean that after the procedure is ∴ x – 18 is odd and less than 82.
carried out 49 times, there is a single number whose value ∴ x – 18 = 34, 33, 32,31, or 30. Choice (5)
is 49 less than the original sum.
∴final number =
(50) (51) – 49 = 1275 – 49 121. There are x players in each team and N pairs of teams
have a common player.
2
∴ total number of players in the 2N + 1 teams
= 1226. Choice (5)
= x(2N + 1) – N = N(2x – 1) + x. Choice (2)
116. 73024 = (74)756 = (2401)756
122. Sum of the odd digits of the number = sum of its even
∴(74) raised to the power of any natural number must
digits = A + B.
end with 01
∴ It is divisible by 11.
∴(74)756 must end with 01. It is a perfect square.
Choice (4) ∴ it must be divisible by 112 i.e., 121 → (1)
Its last two digits are equal.
117. As per the definition, Div(N) always takes a single-digit The tens digit of any odd perfect square is even.
number for its value. Therefore, ‘AABB’ is the square of an even multiple of
We see that any number and the sum of the digits of 11.
that number have the same 9’s remainder (remainder ∴ ‘AABB’ is 442 (i.e. 1764), 662 (i.e. 4356) or 882
rule for 9). In the process of adding up the digit of a (7744).
given number, repeatedly, the 9’s remainder doesn’t ∴ AABB = 882
change. = 7744. So, only 1 possibility. Choice (4)
∴ The number N for which Div(N) = 9, are the
multiples of 9. 123. n(n!) = [(n + 1) – 1]n! = (n + 1)! – n!
600 ∴ 3(3!) = 4! – 3!
There are or 66 multiples of 9 less than 600. 4(4!) = 5! – 4!
9
– –
Choice (3)
15(15!) = 16! – 15!
∴ M = 16! – 3!
and M – 15 = 16! – 6 – 15 = 16! – 21
14
The divisor is 14! – 2 129. Consider the choices.
Consider (14! – 2) (15) (16). It is (7 + 4 3 ) −50 = (7 − 4 3 )50 = 7 – C1 7 ( 4 3 )
50 50 49
(A)
16! – 480
The dividend is 16! – 21 = 16! – 480 + 459 + C2 748 ( 4 3 )2 ……
50

 16! − 21   459  The expression in A is double of this and hence


∴ R   = R  = 459 Choice (C) irrational.
 14! − 2   14! − 2 
(B) (7 − 4 3 ) −50 = (7 + 4 3 )50 = 750 + 50C1 749 ( 4 3 )
124. The divisors and corresponding remainders are + …. The expression in B is double of this and hence
tabulated below irrational.
Divisor 5 6 7 8 Choice (C) is 7 + 4 3 , which is irrational.
Remainder 3 4 5 6 Choice (D)
The required numbers are of the form [LCM (5, 6, 7, 8)]k –2 n n
or 840k – 2. The required numbers are 130. In n is odd, a + b is divisible by a + b.
840(2) – 2 ∴ If there is odd factor in the index n, an + bn can’t be
840(3) – 2 prime for example.
– – – 296 + 1 = (232)3 + 13 is divisible by 232 + 1
840(11) –2 2160 + 1 = (232)5 + 15 is divisible by 232 + 1
There are 10 such 4-digit numbers. Choice (B) 270 + 1 = (210)7 + 17 is divisible by 210 + 1
Choice (D)
1 1
125. x = + + ……. 131. The first sheet provides pages 1, 2, 199, 200. The
22 32
total is 402.
1 1 1 The second sheet provides pages 3, 4, 197, 198. The
Let y = 2 + 2 + 2 + .......
2 4 6 total is 402
1  1 1  1 The total on any sheet is constant, irrespective of n.
= 1 + + + .....  = (1 + x) Choice (D)
22  22 3 2  4
1 1 1 132. u + v + w + x + y = 15
we need + + + ….. or x – y. We need the max values of uvx + uvy + uwx + uwy
2 2
3 5 72
= (uv + uw) (x + y) = u(v + w) (x + y). This occurs
1 3x − 1
x–y=x– (1 + x) = Choice (A) when
4 4 u = v + w = x + y = 5 and it is 5(5) (5). Choice (A)
126. It is convenient to consider a general expression of
 1301 
an + 1 − bn + 1 133. Rem   = 20 ≡ –1
the form .  21 
an + bn Let the given expression be E.
an + 1 − bn + 1 (a − b) (a n + a n + 1b + .... + b n ) E
E= = >a–b Rem   = (–1)10 – (–1) = 2 Choice (B)
a +b
n n
an + bn  21 
49 80 − 42 80
∴ > 49 – 42 i.e. 7 Choice (D) 134. 23 < a < b < 40 < c < d < 50
49 79 + 42 79 There are 16 number between 23 and 40, therefore
16
C2 ways of selecting (a, b)
127. To obtain the 9’s remainder of any number, we can There are 9 numbers between 40 and 50, therefore
9
proceed in a number of ways. C2 ways of selecting (c, d)
At one extreme, we can consider the given number 16(15) 9(8)
16
itself as a single quantity. C2 = = 120 and 9C2 = = 36
As the other extreme, we can focus on each individual 2 2
digit and add them all up. ∴ The number of values of (a, b, c, d) is 120(36)
As an intermediate approach, we can ‘fragment’ the = 4320 Choice (B)
number in any convenient way and consider the
fragments. 135. N = 15a + 7 ….. (1)
In the given question, the third approach is the most = 21b + 10 … (2)
convenient. = 35c + 17 … (3)
i.e. instead of A itself (or the individual digits of A) we (1), (2) ⇒ 15a – 21b = 3
consider B = 7 + 14 + 21 + … + 189 + 196 = 7(1 + 2 + ⇒ 5a – 7b = 1
3 + … + 27 + 28) = 7(14)(29) ⇒ 5a – 5b = 2b + 1
The 9’s remainder of the product of several numbers ⇒ 2b = 5a1 + 4
is the product of the 9’s remainders of the numbers. ⇒ a1 = 2a2
∴ The required remainder is the remainder of 7(5) (2)
a2 = 0 ⇒ a1 = 0 ⇒ b = 2 ⇒ a = 3. ∴ (a, b) = (3, 2),
or 7. Choice (D)
(10, 7) (17, 12) etc.
∴ N = 15a + 7 = 15(3) + 7
128. Let –a = c
or 15(10) + 7 etc
2c + 5b
∴ = (c 2b 5 )1 / 7 i.e. N = K LCM (15, 21) + 52 = 105k + 52
7 when such a number is divided by 35, the remainder
or the AM of c, c, b, b, b, b, b is equal to the GM of the is 17.
7 numbers. ∴ The numbers of the required kind leave a remainder
c +b of 52 when divided by 105. Choice (C)
∴ All the numbers are equal, i.e. b = c and x = =b=c
2
∴2(x – a) + 5(x – b) = 0.
2 5
Choice (D)
15
136. Since there is no restriction that the digits are distinct, 243243
we can consider all the multiples of 6. = Number of factors of
21
99996 − 10002 = Number of factors of 34 × 11 × 13
Number of 5–digit multiples of 6 = +1
6 = (4 + 1) (1 + 1) (1 + 1) = 20 Choice (A)
89994
= + 1 = 15000. These include all the multiples 139. 49 + 52 is divisible by 101 and 5011 + 5111 is
11 11
6
of 12. divisible by 101.
It can be noted that 4911 + 5011 + 5111 + 5211 is an
99996 − 10008 even number. Hence it is divisible 202.
Number of 5–digit multiples of 12 = +1
12 Hence the remainder when 4911 + 5011 + 5111 + 5211 is
= 7500 divided by 202 is 0. Choice (A)
∴ Number of 5-digit multiples of 6 that are not
multiples of 12 = 15000 – 7500 = 7500 140. 256! has (128 + 64 + 32 + 16 + 8 + 4 + 2 + 1) i.e. 255 twos,
Choice (A) (85 + 28 + 9 + 3 + 1) i.e. 126 threes and (51 + 10 + 2) i.e.
63 fives as its factors. When all the zeros at the end of
137. 412 – 1 = 224 – 1 = (212 – 1) (212 + 1) = (26 – 1) (26 + 1) 256! are knocked off, there are no fives left and there
(212 + 1) are (255 – 63) i.e. 192 twos left. To get a 12, 2 twos and
= 7(9) (65) (4097) = 32(5) (7) (13) (17) (241) 1 three are needed. We have 126 threes and 192 twos
∴ 412 – 1 is divisible by 5, 63 and 255. Choice (D) 192
available. So, a maximum of i.e.. 96 twelves are
2
138. 243243 = 243 × 1001 = 3 × 7 × 11 × 13
5
available.
= 21(34 × 11 × 13) ∴ The index of the highest power of 12 in Q is 96 i.e.
∴ The number of factors of 243243 that are divisible 1296 divides Q but 1297 does not. Choice (C)
by 21

16

Das könnte Ihnen auch gefallen